Jump to content

unvhoang1998's Content

There have been 51 items by unvhoang1998 (Search limited from 05-06-2020)



Sort by                Order  

#520758 Topic đăng ký CTV cho thư viện online MathPedia.

Posted by unvhoang1998 on 22-08-2014 - 17:48 in Gặp gỡ Toán học 2014

Tên: Ung Nguyễn Vũ Hoàng THPT chuyên Lê Quý Đôn, Bình Định

Chuyên đề: Hình Học, Đa thức.

Địa chỉ Facebook:   https://www.facebook...ang.ungnguyenvu




#457633 Topic về số học, các bài toán về số học.

Posted by unvhoang1998 on 14-10-2013 - 17:42 in Số học

Bài 57: CMR : Dãy số sau chúa vô hạn các số nguyên đôi một nguyên tố cùng nhau

$t_{n}=\frac{1}{k!}n\left (  n+1\right )\left ( n+k-1 \right )$

với mọi $n,k\in Z^{+}$




#455428 Chứng minh rằng:$cos36^ocos72^o=\frac{1}{4}$

Posted by unvhoang1998 on 05-10-2013 - 21:22 in Công thức lượng giác, hàm số lượng giác

Chứng minh rằng: 

 

c. $tan9^o-tan27^o-tan63^o+tan81^o=4$

câu này giải như sau :

VT $=\frac{\sin90^o}{\cos9^o\cos81^o}-\frac{\sin90^o}{\cos27^o\cos63^o}$

    $=\frac{1}{\cos9^o\sin9^o}-\frac{1}{\cos27^o\sin27^o}$

    $=\frac{2}{\sin18^o}-\frac{2}{\sin54^o}$

    $=\frac{2(\sin54^o-\sin18^o)}{\sin18^o\sin54^o}=\frac{2.2\cos36^o\sin18^o}{\sin18^o\sin54^o}=4$ (Đpcm)




#457796 $ (a+b-c-1)(b+c-a-1)(c+a-b-1) \le 8 $

Posted by unvhoang1998 on 15-10-2013 - 20:33 in Bất đẳng thức - Cực trị

sao chưa ai chém bài mình zậy đành tự xử thôi!!!!!!!!!!!

 từ giả thiết ta có thể đặt: $x=a-1;y=b-1;z=c-1$

khi đó ta được $x,y,z\in[0;1]$ và $x+y+z=1$

nhờ vậy ta được 

$P=(x+1)(y+1)+(y+1)(z+1)+(z+1)(x+1)+(x+1)(y+1)(z+1)=2(xy+yz+zx)+xyz+3(x+y+z)+4$

 $=2(xy+yz+zx)+xyz+7\geq 7$

vậy $MIN P=7$

dấu '=' xảy ra khi $xy+yz+zx=0;xyz=0;x+y+z=1$ hay một trong ba số $a,b,c$ bằng 2 và hai số còn lại bằng 1




#457521 $ (a+b-c-1)(b+c-a-1)(c+a-b-1) \le 8 $

Posted by unvhoang1998 on 13-10-2013 - 22:14 in Bất đẳng thức - Cực trị

Góp cho vui nhá!!!

Bài 18: cho $a,b,c\in[1;2]$ thoả $a+b+c =4$

 Tìm MIN: P=$ab+bc+ca+abc$




#457602 bài giảng số học

Posted by unvhoang1998 on 14-10-2013 - 13:02 in Tài nguyên Olympic toán

thanks nhìu




#457741 Sách của Titu

Posted by unvhoang1998 on 15-10-2013 - 13:03 in Tài nguyên Olympic toán

Mình có vài cuốn của Titu muốn chia sẻ cho mấy bạn ài cần thì cứ tự nhiên




#456104 $a.IA^{2}+b.IB^{2}+c.IC^{2}=abc$

Posted by unvhoang1998 on 08-10-2013 - 14:02 in Hình học phẳng

Cho I là tâm đườn tròn nội tiếp tam giác ABC.a,b,c là các cạnh của tam giác. Chứng minh

$a.IA^{2}+b.IB^{2}+c.IC^{2}=abc$

Cách 1:

 Ta có $a\vec{IA}+b\vec{IB}+c\vec{IC}=0$

           $\Rightarrow (a\vec{IA}+b\vec{IB}+c\vec{IC})^{2}=0$

           $\Rightarrow a^{2}IA^{2}+b^{2}IB^{2}+c^{2}IC^{2}+2ab\vec{IA}.\vec{I B}+2bc\vec{IB}.\vec{IC}+2ca\vec{IC}.\vec{IA}=0$

           $\Rightarrow a^{2}IA^{2}+b^{2}IB^{2}+c^{2}IC^{2}+ab(IA^{2}+IB^{2}-AB^{2})+bc(IB^{2}+IC^{2}-BC^{2})+ca(IC^{2}+IA^{2}-CA^{2})=0$

           $\Rightarrow a^{2}IA^{2}+b^{2}IB^{2}+c^{2}IC^{2}+ab(IA^{2}+IB^{2}-AB^{2})+bc(IB^{2}+IC^{2}-BC^{2})+ca(IC^{2}+IA^{2}-CA^{2})=0$

Vậy ta có dpcm

Cách 2 :Hướng dẫn gọi H, K lllà Hình chiếu của I lên AB và AC dùng công thức hình chiếu




#457805 Đề thi chọn đội tuyển HSG QG tỉnh Hải Phòng năm học 2013-2014

Posted by unvhoang1998 on 15-10-2013 - 21:01 in Thi HSG cấp Tỉnh, Thành phố. Olympic 30-4. Đề thi và kiểm tra đội tuyển các cấp.

cai chỗ $a_n=3^n+3.\left ( -1 \right )^n$ nói dễ chả dẽ tí nào




#458026 Hai tài liệu tốt

Posted by unvhoang1998 on 16-10-2013 - 21:52 in Tài nguyên Olympic toán

Mình muốn share cho các mem hai tài liệu sau >>>>>>>>>>>>>>>>> ai cần cứ tải :icon6:  :icon6:  :icon6:  :icon6:  :icon6:  :icon6:  :icon6:  :icon6:  :icon6:  :icon6:  :icon6:




#458940 Hai tài liệu tốt

Posted by unvhoang1998 on 20-10-2013 - 21:01 in Tài nguyên Olympic toán

àh bởi vì những trang trước hôk thực sự cần thiết cho nên mình hok post lên. Với lại phần kia không thuộc về phần học sơ cấp nhìu nên mình hôk đăng




#457631 chứng minh rằng: nếu p nguyên tố lớn hơn 5 thì (p-2)! -1 không là lũy thừ...

Posted by unvhoang1998 on 14-10-2013 - 17:39 in Số học

Vì $p>5$ nên $p-1$ là ước của $\left ( p-2 \right )!$

Gs tồn tại $p$ sao cho $\left ( p-2 \right )!-1=p^s$

Khi đó ta có $p^s+1 \vdots p-1$

Mặt khác, $p^s-1 \vdots p-1$

Suy ra $2 \vdots p-1$ (mâu thuẫn vì $p>5$)

Suy ra đpcm

vẫn cón một mâu thuẫn tại sao phải  có $p>5$ thay vào đó trong lập luận của bạn cũng chỉ $p>3$ là là đủ để có đk vô lí 

Vậy thiếu sót chỗ nào?????????????????????




#458031 $2^{x}+3^{x} \vdots x^{2}$

Posted by unvhoang1998 on 16-10-2013 - 22:07 in Số học

Chứng Minh Rằng có vô số số nguyên dương $x$ thoả mãn:

  $2^{x}+3^{x}   \vdots   x^{2}$




#457594 CMR $HM$ vuông góc với dây cung chung của $(ABC)$ và...

Posted by unvhoang1998 on 14-10-2013 - 12:06 in Hình học

Cho tam giác $ABC$ nhọn ($AB\neq AC$). gọi $H$ là trực tâm của tam giác, $M$ là trung điểm $BC$. Các điểm $Đ,E$ lần lược thuộc các cạnh $AB,AC$ sao cho $D,H,E$ thẳng hàng. CMR $HM$ vuông góc với dây cung chung của $(ABC)$ và $(ADE)$




#458021 $tan34^{o}$

Posted by unvhoang1998 on 16-10-2013 - 21:41 in Các bài toán Lượng giác khác

Chứng minh rằng:

   $tan34^{o} > \frac{2}{3}$




#457940 $(a+b)^{p}\equiv a^{p}+b^{p} (mod p)...

Posted by unvhoang1998 on 16-10-2013 - 17:30 in Số học

Bài này vẫn còn một cách nữa đó là:

áp dụng công thức sau: $x^{p} \equiv x (mod  p)$ với $p$ nguyên tố 

thay lần lượt $x$ bằng $a,b,a+b$ từ đó ta có đpcm




#512439 $P(2015) \vdots 7$

Posted by unvhoang1998 on 12-07-2014 - 14:18 in Đa thức

Cho $P(x)$ là một đa thức với hệ số nguyên không âm và các hệ số không vượt quá $14$ thỏa mãn điều sau:

$P(15)=491998$. Chứng minh rằng $P(2015) \vdots 7$




#455029 $t_{n}=\frac{1}{k!}n\left (...

Posted by unvhoang1998 on 04-10-2013 - 13:21 in Số học

CMR : Dãy số sau chúa vô hạn các số nguyên đôi một nguyên tố cùng nhau

$t_{n}=\frac{1}{k!}n\left (  n+1\right )\left ( n+k-1 \right )$

với mọi $n,k\in Z^{+}$




#473177 CMR $SC=SP$ khi và chỉ khi $MK=ML$

Posted by unvhoang1998 on 27-12-2013 - 13:24 in Hình học

==================================================================================================

Gọi $I,J$ là giao của $SP$ với $(O)$ (như hình vẽ)

có ngay $IK=LJ$

mà góc $SCM=SPC$ nên $MI=MJ$

Do đó $MK=ML$

 

làm phiền bạn có thể giải thích cho mình chỗ này được không




#471712 CMR $SC=SP$ khi và chỉ khi $MK=ML$

Posted by unvhoang1998 on 19-12-2013 - 12:41 in Hình học


Cho điểm $P$ nằm trong tam giác $ABC$. Các tia $AP,BP,CP$ lần lượt cắt đường tròn ngoại tiếp tam giác tại $K,L,M$.

Tiếp tuyến tại $C$ của đường tròn cắt $AB$ tại $S$.

CMR $SC=SP$ khi và chỉ khi $MK=ML$




#474711 Chứng minh rằng $KP=KQ$

Posted by unvhoang1998 on 02-01-2014 - 13:07 in Hình học

Cho tam giác $ABC$. $AK$ là đường đối trung của góc $BAC$ của tam giác ($K$ thuộc $BC$). Đường tròn ngoại tiếp tam giác $AKC$ cắt $AB$ tại $P$. Đường tròn ngoại tiếp tam giác $AKB$ cắt $AC$ tại $Q$. Chứng minh rằng $KP=KQ$




#503962 $f\left ( x^{2}+y^{2}+2f\left ( xy \r...

Posted by unvhoang1998 on 04-06-2014 - 14:53 in Phương trình hàm

Theo mình thì mình giải bài này với điều kiện b được thay bằng điều kiện là $f$ là hàm đơn ánh trên $R$ và mình giải như sau:

Đầu tiên ta cho $x=0 ; y=0$ khi đó ta được

$f(2f(0))=f^{2}(0)$   Đặt $f(0)=a$

Cho $ y=-x4 thì ta được

$f(2x^{2}+2f^{2}(-x^{2})=f^{2}(0)=f(2f(0))  ,\forall x \in R $ 

Theo tính chất đơn ánh thì 

$x^{2}+f(-x^{2})=f(0)  , \forall x \in R$

$\Leftrightarrow f(x)=x+a  , \forall x \leq 0$

Cho $ x=y$

thì $f(2x^{2}+2f(2x^{2}))=f^{2}(2x)   , \forall x \in R$

lại cho $x \rightarrow 2x ,   y \rightarrow 0$

thì $f(4x^{2}+2a)=f^{2}(2x)   , \forall x \in R$

do vậy nên 

$ f(x^{2}) +x^{2} =2x^{2} +a  ,   \forall x \in R$

$\Leftrightarrow f(x) =x +a ,   \forall x \geq 0$

Tóm lại $f(x) =x +a,   \forall x \in R$

Thử lại thấy $a=0$ 

Vậy hàm số duy nhất thoả mãn là 

$f(x) =x ,  \forall x \in R$




#471911 Tìm tất cả các bộ $(A,B)$

Posted by unvhoang1998 on 20-12-2013 - 18:34 in Tổ hợp và rời rạc

ban có thể giải thích rõ làm s để có được kết quả đó hay không (nêu rõ cách tính)  làm sao để có được $(n-1).n.4^{n-2}$




#471736 Tìm tất cả các bộ $(A,B)$

Posted by unvhoang1998 on 19-12-2013 - 17:02 in Tổ hợp và rời rạc

Cho tập hợp $X= { 1,2,...,n } $. Gọi $A,B$ là hai tập con của $X$. Tìm tất cả các bộ $(A,B)$ thỏa mãn $A$ không phải là tập con của $B$ và $B$ cũng không phải là tập con của $A$




#457514 $a_{1}\geq a_{2}\geq a_{3}\...

Posted by unvhoang1998 on 13-10-2013 - 21:49 in Bất đẳng thức - Cực trị

Bất đẳng thức dạng cơ bản nhất:

1) Cho $a_{1}\geq a_{2}\geq a_{3}\geq...........\geq a_{2012}>0$ thoả mãn:

 $a_{1}+ a_{2}+ a_{3} \leq 2012$ và

 $a_{4}+ a_{5}+ a_{6} +...........+ a_{2012}\leq 2012$

Tìm MAX: P=$\sum_{i=1}^{2012}a_{i}^2$

2)  Cho $a_{1}\geq a_{2}\geq a_{3}\geq...........\geq a_{100}>0$ thoả mãn

 $a_{1}^{2}+ a_{2}^{2} \geq 100$ và

 $a_{3}^{2}+ a_{4}^{2}+ a_{5}^{2} +...........+ a_{100}^{2}\geq 100$

Tìm MIN: Q=$\sum_{i=1}^{100}a_{i}$

Nhìn có vẻ ta thấy rằng bài 2 là bài toán ngược của bài 1